Mathematical and Physical Journal
for High Schools
Issued by the MATFUND Foundation
Already signed up?
New to KöMaL?

Problem B. 4184. (May 2009)

B. 4184. The vertices of a cyclic quadrilateral divide the circumscribed circle into four arcs. The midpoints of the arcs are, in this order, F1, F2, F3 and F4. Show that the line segment F1F3 is perpendicular to F2F4.

(3 pont)

Deadline expired on June 15, 2009.


Sorry, the solution is available only in Hungarian. Google translation

Megoldás. A sokszög csúcsai legyenek A,B,C,D az ábrán látható módon, az átlók metszéspontja legyen M. Az egymástól páronként diszjunkt AB,BC,CD,DA ívekhez tartozó középponti szögeket jelölje rendre \alpha,\beta,\gamma,\delta, ezek összege nyilván 2\pi.

Az F2 pontból az AB ív \alpha/2 szög alatt látszik, ezért az AF1 ív \alpha/4 szög alatt látszik. Az AF2F4 szög nagysága hasonlóképpen \delta/4, vagyis az F1F2F4 szög nagysága (\alpha+\delta)/4. Ugyanígy számolhatjuk ki azt is, hogy az F2F1F3 szög nagysága (\beta+\gamma)/4. Ezek alapján az F1F2M háromszögben az F1 és F2 csúcsoknál lévő szögek összege (\alpha+\beta+\gamma+\delta)/4=\pi/2, tehát az M csúcsnál lévő szög valóban derékszög.


Statistics:

75 students sent a solution.
3 points:53 students.
2 points:15 students.
1 point:2 students.
0 point:1 student.
Unfair, not evaluated:4 solutionss.

Problems in Mathematics of KöMaL, May 2009